N-movers: How much of the infinite board can I reach?












36












$begingroup$


Single moves



The board is an infinite 2 dimensional square grid, like a limitless chess board. A piece with value N (an N-mover) can move to any square that is a distance of exactly the square root of N from its current square (Euclidean distance measured centre to centre).



For example:




  • A 1-mover can move to any square that is horizontally or vertically adjacent

  • A 2-mover can move to any square that is diagonally adjacent

  • A 5-mover moves like a chess knight


Note that not all N-movers can move. A 3-mover can never leave its current square because none of the squares on the board are a distance of exactly root 3 from the current square.



Multiple moves



If allowed to move repeatedly, some pieces can reach any square on the board. For example, a 1-mover and a 5-mover can both do this. A 2-mover can only move diagonally and can only reach half of the squares. A piece that cannot move, like a 3-mover, cannot reach any of the squares (the starting square is not counted as "reached" if no movement occurs).



1-mover2-mover3-mover4-mover5-mover8-mover9-mover10-mover20-mover25-mover40-mover64-mover65-mover68-mover



The images show which squares can be reached. More details on hover. Click for larger image.




  • Squares reachable in 1 or more moves are marked in black

  • Squares reachable in exactly 1 move are shown by red pieces
    (apart from the 3-mover, which cannot move)


What proportion of the board can a given N-mover reach?



Input




  • A positive integer N


Output




  • The proportion of the board that an N-mover can reach

  • This is a number from 0 to 1 (both inclusive)

  • For this challenge, output as a fraction in lowest terms, like 1/4, is allowed


So for input 8, both 1/8 and 0.125 are acceptable outputs.



Scoring



This is code golf. The score is the length of the code in bytes. For each language, the shortest code wins.



Test cases



In the format input : output as fraction : output as decimal



  1 : 1     : 1
2 : 1/2 : 0.5
3 : 0 : 0
4 : 1/4 : 0.25
5 : 1 : 1
6 : 0 : 0
7 : 0 : 0
8 : 1/8 : 0.125
9 : 1/9 : 0.1111111111111111111111111111
10 : 1/2 : 0.5
13 : 1 : 1
16 : 1/16 : 0.0625
18 : 1/18 : 0.05555555555555555555555555556
20 : 1/4 : 0.25
25 : 1 : 1
26 : 1/2 : 0.5
64 : 1/64 : 0.015625
65 : 1 : 1
72 : 1/72 : 0.01388888888888888888888888889
73 : 1 : 1
74 : 1/2 : 0.5
80 : 1/16 : 0.0625
81 : 1/81 : 0.01234567901234567901234567901
82 : 1/2 : 0.5
144 : 1/144 : 0.006944444444444444444444444444
145 : 1 : 1
146 : 1/2 : 0.5
148 : 1/4 : 0.25
153 : 1/9 : 0.1111111111111111111111111111
160 : 1/32 : 0.03125
161 : 0 : 0
162 : 1/162 : 0.006172839506172839506172839506
163 : 0 : 0
164 : 1/4 : 0.25
241 : 1 : 1
242 : 1/242 : 0.004132231404958677685950413223
244 : 1/4 : 0.25
245 : 1/49 : 0.02040816326530612244897959184
260 : 1/4 : 0.25
261 : 1/9 : 0.1111111111111111111111111111
288 : 1/288 : 0.003472222222222222222222222222
290 : 1/2 : 0.5
292 : 1/4 : 0.25
293 : 1 : 1
324 : 1/324 : 0.003086419753086419753086419753
325 : 1 : 1
326 : 0 : 0
360 : 1/72 : 0.01388888888888888888888888889
361 : 1/361 : 0.002770083102493074792243767313
362 : 1/2 : 0.5
369 : 1/9 : 0.1111111111111111111111111111
370 : 1/2 : 0.5
449 : 1 : 1
450 : 1/18 : 0.05555555555555555555555555556
488 : 1/8 : 0.125
489 : 0 : 0
490 : 1/98 : 0.01020408163265306122448979592
520 : 1/8 : 0.125
521 : 1 : 1
522 : 1/18 : 0.05555555555555555555555555556
544 : 1/32 : 0.03125
548 : 1/4 : 0.25
549 : 1/9 : 0.1111111111111111111111111111
584 : 1/8 : 0.125
585 : 1/9 : 0.1111111111111111111111111111
586 : 1/2 : 0.5
592 : 1/16 : 0.0625
593 : 1 : 1
596 : 1/4 : 0.25
605 : 1/121 : 0.008264462809917355371900826446
610 : 1/2 : 0.5
611 : 0 : 0
612 : 1/36 : 0.02777777777777777777777777778
613 : 1 : 1
624 : 0 : 0
625 : 1 : 1









share|improve this question











$endgroup$








  • 8




    $begingroup$
    I posted this question on Math.SE: math.stackexchange.com/questions/3108324/…
    $endgroup$
    – infmagic2047
    Feb 11 at 4:27










  • $begingroup$
    Interesting conjecture!
    $endgroup$
    – trichoplax
    Feb 11 at 10:06






  • 1




    $begingroup$
    "A piece that cannot move, like a 3-mover, cannot reach any of the squares". Interestingly enough, even if you count the starting square, since the board is infinite, it still converges to 0 as a proportion.
    $endgroup$
    – Beefster
    Feb 11 at 17:10










  • $begingroup$
    @Beefster good point. I went with this way to make the limit easier to find without having to go all the way to infinity...
    $endgroup$
    – trichoplax
    Feb 11 at 18:13






  • 1




    $begingroup$
    @infmagic2047 's math.se question about the prime factoring approach now has an answer with a complete proof.
    $endgroup$
    – Ørjan Johansen
    Feb 13 at 18:10
















36












$begingroup$


Single moves



The board is an infinite 2 dimensional square grid, like a limitless chess board. A piece with value N (an N-mover) can move to any square that is a distance of exactly the square root of N from its current square (Euclidean distance measured centre to centre).



For example:




  • A 1-mover can move to any square that is horizontally or vertically adjacent

  • A 2-mover can move to any square that is diagonally adjacent

  • A 5-mover moves like a chess knight


Note that not all N-movers can move. A 3-mover can never leave its current square because none of the squares on the board are a distance of exactly root 3 from the current square.



Multiple moves



If allowed to move repeatedly, some pieces can reach any square on the board. For example, a 1-mover and a 5-mover can both do this. A 2-mover can only move diagonally and can only reach half of the squares. A piece that cannot move, like a 3-mover, cannot reach any of the squares (the starting square is not counted as "reached" if no movement occurs).



1-mover2-mover3-mover4-mover5-mover8-mover9-mover10-mover20-mover25-mover40-mover64-mover65-mover68-mover



The images show which squares can be reached. More details on hover. Click for larger image.




  • Squares reachable in 1 or more moves are marked in black

  • Squares reachable in exactly 1 move are shown by red pieces
    (apart from the 3-mover, which cannot move)


What proportion of the board can a given N-mover reach?



Input




  • A positive integer N


Output




  • The proportion of the board that an N-mover can reach

  • This is a number from 0 to 1 (both inclusive)

  • For this challenge, output as a fraction in lowest terms, like 1/4, is allowed


So for input 8, both 1/8 and 0.125 are acceptable outputs.



Scoring



This is code golf. The score is the length of the code in bytes. For each language, the shortest code wins.



Test cases



In the format input : output as fraction : output as decimal



  1 : 1     : 1
2 : 1/2 : 0.5
3 : 0 : 0
4 : 1/4 : 0.25
5 : 1 : 1
6 : 0 : 0
7 : 0 : 0
8 : 1/8 : 0.125
9 : 1/9 : 0.1111111111111111111111111111
10 : 1/2 : 0.5
13 : 1 : 1
16 : 1/16 : 0.0625
18 : 1/18 : 0.05555555555555555555555555556
20 : 1/4 : 0.25
25 : 1 : 1
26 : 1/2 : 0.5
64 : 1/64 : 0.015625
65 : 1 : 1
72 : 1/72 : 0.01388888888888888888888888889
73 : 1 : 1
74 : 1/2 : 0.5
80 : 1/16 : 0.0625
81 : 1/81 : 0.01234567901234567901234567901
82 : 1/2 : 0.5
144 : 1/144 : 0.006944444444444444444444444444
145 : 1 : 1
146 : 1/2 : 0.5
148 : 1/4 : 0.25
153 : 1/9 : 0.1111111111111111111111111111
160 : 1/32 : 0.03125
161 : 0 : 0
162 : 1/162 : 0.006172839506172839506172839506
163 : 0 : 0
164 : 1/4 : 0.25
241 : 1 : 1
242 : 1/242 : 0.004132231404958677685950413223
244 : 1/4 : 0.25
245 : 1/49 : 0.02040816326530612244897959184
260 : 1/4 : 0.25
261 : 1/9 : 0.1111111111111111111111111111
288 : 1/288 : 0.003472222222222222222222222222
290 : 1/2 : 0.5
292 : 1/4 : 0.25
293 : 1 : 1
324 : 1/324 : 0.003086419753086419753086419753
325 : 1 : 1
326 : 0 : 0
360 : 1/72 : 0.01388888888888888888888888889
361 : 1/361 : 0.002770083102493074792243767313
362 : 1/2 : 0.5
369 : 1/9 : 0.1111111111111111111111111111
370 : 1/2 : 0.5
449 : 1 : 1
450 : 1/18 : 0.05555555555555555555555555556
488 : 1/8 : 0.125
489 : 0 : 0
490 : 1/98 : 0.01020408163265306122448979592
520 : 1/8 : 0.125
521 : 1 : 1
522 : 1/18 : 0.05555555555555555555555555556
544 : 1/32 : 0.03125
548 : 1/4 : 0.25
549 : 1/9 : 0.1111111111111111111111111111
584 : 1/8 : 0.125
585 : 1/9 : 0.1111111111111111111111111111
586 : 1/2 : 0.5
592 : 1/16 : 0.0625
593 : 1 : 1
596 : 1/4 : 0.25
605 : 1/121 : 0.008264462809917355371900826446
610 : 1/2 : 0.5
611 : 0 : 0
612 : 1/36 : 0.02777777777777777777777777778
613 : 1 : 1
624 : 0 : 0
625 : 1 : 1









share|improve this question











$endgroup$








  • 8




    $begingroup$
    I posted this question on Math.SE: math.stackexchange.com/questions/3108324/…
    $endgroup$
    – infmagic2047
    Feb 11 at 4:27










  • $begingroup$
    Interesting conjecture!
    $endgroup$
    – trichoplax
    Feb 11 at 10:06






  • 1




    $begingroup$
    "A piece that cannot move, like a 3-mover, cannot reach any of the squares". Interestingly enough, even if you count the starting square, since the board is infinite, it still converges to 0 as a proportion.
    $endgroup$
    – Beefster
    Feb 11 at 17:10










  • $begingroup$
    @Beefster good point. I went with this way to make the limit easier to find without having to go all the way to infinity...
    $endgroup$
    – trichoplax
    Feb 11 at 18:13






  • 1




    $begingroup$
    @infmagic2047 's math.se question about the prime factoring approach now has an answer with a complete proof.
    $endgroup$
    – Ørjan Johansen
    Feb 13 at 18:10














36












36








36


5



$begingroup$


Single moves



The board is an infinite 2 dimensional square grid, like a limitless chess board. A piece with value N (an N-mover) can move to any square that is a distance of exactly the square root of N from its current square (Euclidean distance measured centre to centre).



For example:




  • A 1-mover can move to any square that is horizontally or vertically adjacent

  • A 2-mover can move to any square that is diagonally adjacent

  • A 5-mover moves like a chess knight


Note that not all N-movers can move. A 3-mover can never leave its current square because none of the squares on the board are a distance of exactly root 3 from the current square.



Multiple moves



If allowed to move repeatedly, some pieces can reach any square on the board. For example, a 1-mover and a 5-mover can both do this. A 2-mover can only move diagonally and can only reach half of the squares. A piece that cannot move, like a 3-mover, cannot reach any of the squares (the starting square is not counted as "reached" if no movement occurs).



1-mover2-mover3-mover4-mover5-mover8-mover9-mover10-mover20-mover25-mover40-mover64-mover65-mover68-mover



The images show which squares can be reached. More details on hover. Click for larger image.




  • Squares reachable in 1 or more moves are marked in black

  • Squares reachable in exactly 1 move are shown by red pieces
    (apart from the 3-mover, which cannot move)


What proportion of the board can a given N-mover reach?



Input




  • A positive integer N


Output




  • The proportion of the board that an N-mover can reach

  • This is a number from 0 to 1 (both inclusive)

  • For this challenge, output as a fraction in lowest terms, like 1/4, is allowed


So for input 8, both 1/8 and 0.125 are acceptable outputs.



Scoring



This is code golf. The score is the length of the code in bytes. For each language, the shortest code wins.



Test cases



In the format input : output as fraction : output as decimal



  1 : 1     : 1
2 : 1/2 : 0.5
3 : 0 : 0
4 : 1/4 : 0.25
5 : 1 : 1
6 : 0 : 0
7 : 0 : 0
8 : 1/8 : 0.125
9 : 1/9 : 0.1111111111111111111111111111
10 : 1/2 : 0.5
13 : 1 : 1
16 : 1/16 : 0.0625
18 : 1/18 : 0.05555555555555555555555555556
20 : 1/4 : 0.25
25 : 1 : 1
26 : 1/2 : 0.5
64 : 1/64 : 0.015625
65 : 1 : 1
72 : 1/72 : 0.01388888888888888888888888889
73 : 1 : 1
74 : 1/2 : 0.5
80 : 1/16 : 0.0625
81 : 1/81 : 0.01234567901234567901234567901
82 : 1/2 : 0.5
144 : 1/144 : 0.006944444444444444444444444444
145 : 1 : 1
146 : 1/2 : 0.5
148 : 1/4 : 0.25
153 : 1/9 : 0.1111111111111111111111111111
160 : 1/32 : 0.03125
161 : 0 : 0
162 : 1/162 : 0.006172839506172839506172839506
163 : 0 : 0
164 : 1/4 : 0.25
241 : 1 : 1
242 : 1/242 : 0.004132231404958677685950413223
244 : 1/4 : 0.25
245 : 1/49 : 0.02040816326530612244897959184
260 : 1/4 : 0.25
261 : 1/9 : 0.1111111111111111111111111111
288 : 1/288 : 0.003472222222222222222222222222
290 : 1/2 : 0.5
292 : 1/4 : 0.25
293 : 1 : 1
324 : 1/324 : 0.003086419753086419753086419753
325 : 1 : 1
326 : 0 : 0
360 : 1/72 : 0.01388888888888888888888888889
361 : 1/361 : 0.002770083102493074792243767313
362 : 1/2 : 0.5
369 : 1/9 : 0.1111111111111111111111111111
370 : 1/2 : 0.5
449 : 1 : 1
450 : 1/18 : 0.05555555555555555555555555556
488 : 1/8 : 0.125
489 : 0 : 0
490 : 1/98 : 0.01020408163265306122448979592
520 : 1/8 : 0.125
521 : 1 : 1
522 : 1/18 : 0.05555555555555555555555555556
544 : 1/32 : 0.03125
548 : 1/4 : 0.25
549 : 1/9 : 0.1111111111111111111111111111
584 : 1/8 : 0.125
585 : 1/9 : 0.1111111111111111111111111111
586 : 1/2 : 0.5
592 : 1/16 : 0.0625
593 : 1 : 1
596 : 1/4 : 0.25
605 : 1/121 : 0.008264462809917355371900826446
610 : 1/2 : 0.5
611 : 0 : 0
612 : 1/36 : 0.02777777777777777777777777778
613 : 1 : 1
624 : 0 : 0
625 : 1 : 1









share|improve this question











$endgroup$




Single moves



The board is an infinite 2 dimensional square grid, like a limitless chess board. A piece with value N (an N-mover) can move to any square that is a distance of exactly the square root of N from its current square (Euclidean distance measured centre to centre).



For example:




  • A 1-mover can move to any square that is horizontally or vertically adjacent

  • A 2-mover can move to any square that is diagonally adjacent

  • A 5-mover moves like a chess knight


Note that not all N-movers can move. A 3-mover can never leave its current square because none of the squares on the board are a distance of exactly root 3 from the current square.



Multiple moves



If allowed to move repeatedly, some pieces can reach any square on the board. For example, a 1-mover and a 5-mover can both do this. A 2-mover can only move diagonally and can only reach half of the squares. A piece that cannot move, like a 3-mover, cannot reach any of the squares (the starting square is not counted as "reached" if no movement occurs).



1-mover2-mover3-mover4-mover5-mover8-mover9-mover10-mover20-mover25-mover40-mover64-mover65-mover68-mover



The images show which squares can be reached. More details on hover. Click for larger image.




  • Squares reachable in 1 or more moves are marked in black

  • Squares reachable in exactly 1 move are shown by red pieces
    (apart from the 3-mover, which cannot move)


What proportion of the board can a given N-mover reach?



Input




  • A positive integer N


Output




  • The proportion of the board that an N-mover can reach

  • This is a number from 0 to 1 (both inclusive)

  • For this challenge, output as a fraction in lowest terms, like 1/4, is allowed


So for input 8, both 1/8 and 0.125 are acceptable outputs.



Scoring



This is code golf. The score is the length of the code in bytes. For each language, the shortest code wins.



Test cases



In the format input : output as fraction : output as decimal



  1 : 1     : 1
2 : 1/2 : 0.5
3 : 0 : 0
4 : 1/4 : 0.25
5 : 1 : 1
6 : 0 : 0
7 : 0 : 0
8 : 1/8 : 0.125
9 : 1/9 : 0.1111111111111111111111111111
10 : 1/2 : 0.5
13 : 1 : 1
16 : 1/16 : 0.0625
18 : 1/18 : 0.05555555555555555555555555556
20 : 1/4 : 0.25
25 : 1 : 1
26 : 1/2 : 0.5
64 : 1/64 : 0.015625
65 : 1 : 1
72 : 1/72 : 0.01388888888888888888888888889
73 : 1 : 1
74 : 1/2 : 0.5
80 : 1/16 : 0.0625
81 : 1/81 : 0.01234567901234567901234567901
82 : 1/2 : 0.5
144 : 1/144 : 0.006944444444444444444444444444
145 : 1 : 1
146 : 1/2 : 0.5
148 : 1/4 : 0.25
153 : 1/9 : 0.1111111111111111111111111111
160 : 1/32 : 0.03125
161 : 0 : 0
162 : 1/162 : 0.006172839506172839506172839506
163 : 0 : 0
164 : 1/4 : 0.25
241 : 1 : 1
242 : 1/242 : 0.004132231404958677685950413223
244 : 1/4 : 0.25
245 : 1/49 : 0.02040816326530612244897959184
260 : 1/4 : 0.25
261 : 1/9 : 0.1111111111111111111111111111
288 : 1/288 : 0.003472222222222222222222222222
290 : 1/2 : 0.5
292 : 1/4 : 0.25
293 : 1 : 1
324 : 1/324 : 0.003086419753086419753086419753
325 : 1 : 1
326 : 0 : 0
360 : 1/72 : 0.01388888888888888888888888889
361 : 1/361 : 0.002770083102493074792243767313
362 : 1/2 : 0.5
369 : 1/9 : 0.1111111111111111111111111111
370 : 1/2 : 0.5
449 : 1 : 1
450 : 1/18 : 0.05555555555555555555555555556
488 : 1/8 : 0.125
489 : 0 : 0
490 : 1/98 : 0.01020408163265306122448979592
520 : 1/8 : 0.125
521 : 1 : 1
522 : 1/18 : 0.05555555555555555555555555556
544 : 1/32 : 0.03125
548 : 1/4 : 0.25
549 : 1/9 : 0.1111111111111111111111111111
584 : 1/8 : 0.125
585 : 1/9 : 0.1111111111111111111111111111
586 : 1/2 : 0.5
592 : 1/16 : 0.0625
593 : 1 : 1
596 : 1/4 : 0.25
605 : 1/121 : 0.008264462809917355371900826446
610 : 1/2 : 0.5
611 : 0 : 0
612 : 1/36 : 0.02777777777777777777777777778
613 : 1 : 1
624 : 0 : 0
625 : 1 : 1






code-golf grid chess board-game






share|improve this question















share|improve this question













share|improve this question




share|improve this question








edited Feb 11 at 1:15







trichoplax

















asked Feb 10 at 21:36









trichoplaxtrichoplax

7,44664076




7,44664076








  • 8




    $begingroup$
    I posted this question on Math.SE: math.stackexchange.com/questions/3108324/…
    $endgroup$
    – infmagic2047
    Feb 11 at 4:27










  • $begingroup$
    Interesting conjecture!
    $endgroup$
    – trichoplax
    Feb 11 at 10:06






  • 1




    $begingroup$
    "A piece that cannot move, like a 3-mover, cannot reach any of the squares". Interestingly enough, even if you count the starting square, since the board is infinite, it still converges to 0 as a proportion.
    $endgroup$
    – Beefster
    Feb 11 at 17:10










  • $begingroup$
    @Beefster good point. I went with this way to make the limit easier to find without having to go all the way to infinity...
    $endgroup$
    – trichoplax
    Feb 11 at 18:13






  • 1




    $begingroup$
    @infmagic2047 's math.se question about the prime factoring approach now has an answer with a complete proof.
    $endgroup$
    – Ørjan Johansen
    Feb 13 at 18:10














  • 8




    $begingroup$
    I posted this question on Math.SE: math.stackexchange.com/questions/3108324/…
    $endgroup$
    – infmagic2047
    Feb 11 at 4:27










  • $begingroup$
    Interesting conjecture!
    $endgroup$
    – trichoplax
    Feb 11 at 10:06






  • 1




    $begingroup$
    "A piece that cannot move, like a 3-mover, cannot reach any of the squares". Interestingly enough, even if you count the starting square, since the board is infinite, it still converges to 0 as a proportion.
    $endgroup$
    – Beefster
    Feb 11 at 17:10










  • $begingroup$
    @Beefster good point. I went with this way to make the limit easier to find without having to go all the way to infinity...
    $endgroup$
    – trichoplax
    Feb 11 at 18:13






  • 1




    $begingroup$
    @infmagic2047 's math.se question about the prime factoring approach now has an answer with a complete proof.
    $endgroup$
    – Ørjan Johansen
    Feb 13 at 18:10








8




8




$begingroup$
I posted this question on Math.SE: math.stackexchange.com/questions/3108324/…
$endgroup$
– infmagic2047
Feb 11 at 4:27




$begingroup$
I posted this question on Math.SE: math.stackexchange.com/questions/3108324/…
$endgroup$
– infmagic2047
Feb 11 at 4:27












$begingroup$
Interesting conjecture!
$endgroup$
– trichoplax
Feb 11 at 10:06




$begingroup$
Interesting conjecture!
$endgroup$
– trichoplax
Feb 11 at 10:06




1




1




$begingroup$
"A piece that cannot move, like a 3-mover, cannot reach any of the squares". Interestingly enough, even if you count the starting square, since the board is infinite, it still converges to 0 as a proportion.
$endgroup$
– Beefster
Feb 11 at 17:10




$begingroup$
"A piece that cannot move, like a 3-mover, cannot reach any of the squares". Interestingly enough, even if you count the starting square, since the board is infinite, it still converges to 0 as a proportion.
$endgroup$
– Beefster
Feb 11 at 17:10












$begingroup$
@Beefster good point. I went with this way to make the limit easier to find without having to go all the way to infinity...
$endgroup$
– trichoplax
Feb 11 at 18:13




$begingroup$
@Beefster good point. I went with this way to make the limit easier to find without having to go all the way to infinity...
$endgroup$
– trichoplax
Feb 11 at 18:13




1




1




$begingroup$
@infmagic2047 's math.se question about the prime factoring approach now has an answer with a complete proof.
$endgroup$
– Ørjan Johansen
Feb 13 at 18:10




$begingroup$
@infmagic2047 's math.se question about the prime factoring approach now has an answer with a complete proof.
$endgroup$
– Ørjan Johansen
Feb 13 at 18:10










6 Answers
6






active

oldest

votes


















16












$begingroup$


JavaScript (Node.js), 144 138 125 74 73 70 bytes





f=(x,n=2,c=0)=>x%n?x-!c?f(x,n+1)/(n%4>2?n/=~c&1:n%4)**c:1:f(x/n,n,c+1)


Try it online!



-4 byte thanks @Arnauld!



Original approach, 125 bytes





a=>(F=(x,n=2)=>n*n>x?[x,0]:x%n?F(x,n+1):[n,...F(x/n,n)])(a).map(y=>r-y?(z*=[,1,.5,p%2?0:1/r][r%4]**p,r=y,p=1):p++,z=r=p=1)&&z


Try it online!



Inspired by the video Pi hiding in prime regularities by 3Blue1Brown.



For each prime factor $p^n$ in the factorization of the number, calculate $f(p^n)$:




  • If $n$ is odd and $pequiv 3text{ (mod 4)}$ - $f(p^n)=0$. Because there is no place to go.

  • If $n$ is even and $pequiv 3text{ (mod 4)}$ - $f(p^n)=frac{1}{p^n}$.

  • If $p=2$ - $f(2^n)=frac{1}{2^n}$.

  • If $pequiv 1text{ (mod 4)}$ - $f(p^n)=1$.


Multiply all those function values, there we are.



Update



Thanks to the effort of contributors from Math.SE, the algorithm is now backed by a proof






share|improve this answer











$endgroup$













  • $begingroup$
    Does the video contain a proof? I've been trying to prove this result for a few hours now but I couldn't figure it out.
    $endgroup$
    – infmagic2047
    Feb 11 at 3:38






  • 1




    $begingroup$
    @infmagic2047 Not really, but it gives a method to count the number of points on a $sqrt{n}$ circle. This is useful when coming down to how the n-mover can go.
    $endgroup$
    – Shieru Asakoto
    Feb 11 at 3:39








  • 3




    $begingroup$
    @infmagic2047 I think it's trivial to prove the cases for $q=prod_{pinmathbb{P}land pin{2,3}text{ (mod 4)}}p^{e_p}$ but the cases for the remaining ones are quite hard to prove formally...
    $endgroup$
    – Shieru Asakoto
    Feb 11 at 11:30








  • 1




    $begingroup$
    @infmagic2047 's math.se question about this approach now has an answer with a complete proof.
    $endgroup$
    – Ørjan Johansen
    Feb 13 at 18:09



















11












$begingroup$


Clean, 189 185 172 171 bytes



import StdEnv
$n#r=[~n..n]
#p=[[x,y]\x<-r,y<-r|x^2+y^2==n]
=sum[1.0\_<-iter n(q=removeDup[k\[a,b]<-[[0,0]:p],[u,v]<-q,k<-[[a+u,b+v]]|all(e=n>=e&&e>0)k])p]/toReal(n^2)


Try it online!



Finds every position reachable in the n-side-length square cornered on the origin in the first quadrant, then divides by n^2 to get the portion of all cells reachable.



This works because:




  • The entire reachable plane can be considered as overlapping copies of this n-side-length square, each cornered on a reachable point from the origin as if it were the origin.

  • All movements come in groups of four with signs ++ +- -+ --, allowing the overlapping tiling to be extended through the other three quadrants by mirroring and rotation.






share|improve this answer











$endgroup$













  • $begingroup$
    My apologies - I was looking at the test cases which go from N=10 to N=13, whereas your test cases include N=11 and N=12 too. You are indeed correct for N=13. +1 from me :)
    $endgroup$
    – trichoplax
    Feb 11 at 0:51






  • 1




    $begingroup$
    @trichoplax I've changed the tests to correspond to the question to avoid the same confusion again
    $endgroup$
    – Οurous
    Feb 11 at 0:53










  • $begingroup$
    I've further tested up to N=145 and all are correct. I couldn't test 146 on TIO due to the 60 second timeout though. I'm expecting some very long run times in answers here...
    $endgroup$
    – trichoplax
    Feb 11 at 1:01






  • 1




    $begingroup$
    Since I took a while to realize this: The reason why the square corners are reachable if there is at least one move (a,b), is the complex equation (a+bi)(a-bi)=a^2+b^2, which in vector form becomes (N,0)=a(a,b)+b(b,-a).
    $endgroup$
    – Ørjan Johansen
    Feb 11 at 17:19



















8












$begingroup$

Mathematica, 80 bytes



d[n_]:=If[#=={},0,1/Det@LatticeReduce@#]&@Select[Tuples[Range[-n,n],2],#.#==n&];


This code is mostly reliant on a mathematical theorem. The basic idea is that the code asks for the density of a lattice given some generating set.



More precisely, we are given some collection of vectors - namely, those whose length squared is N - and asked to compute the density of the set of possible sums of these vectors, compared to all integer vectors. The math at play is that we can always find two vectors (and their opposite) that "generate" (i.e. whose sums are) the same set as the original collection. LatticeReduce does exactly that.



If you have just two vectors, you can imagine drawing an identical parallelogram centered at each reachable point, but whose edge lengths are the given vectors, such that the plane is completely tiled by these parallelograms. (Imagine, for instance, a lattice of "diamond" shapes for n=2). The area of each parallelogram is the determinant of the two generating vectors. The desired proportion of the plane is the reciprocal of this area, since each parallelogram has just one reachable point in it.



The code is a fairly straightforward implementation: Generate the vectors, use LatticeReduce, take the determinant, then take the reciprocal. (It can probably be better golfed, though)






share|improve this answer











$endgroup$













  • $begingroup$
    76 bytes: d@n_:=Boole[#!={}]/Det@LatticeReduce@#&@Select[Range[-n,n]~Tuples~2,#.#==n&]
    $endgroup$
    – lastresort
    Feb 11 at 6:00



















3












$begingroup$


Jelly,  25  24 bytes



ÆFµ%4,2CḄ:3+2Ịị,*/ʋ÷*/)P


A monadic link using the prime factor route.



Try it online!



How?



ÆFµ%4,2CḄ:3+2Ịị,*/ʋ÷*/)P - Link: integer, n               e.g. 11250
ÆF - prime factor, exponent pairs [[2,1], [3,2], [5,4]]
µ ) - for each pair [F,E]:
4,2 - literal list [4,2]
% - modulo (vectorises) [2,1] [3,0] [1,0]
C - complement (1-x) [-1,0] [-2,1] [0,1]
Ḅ - from base 2 -2 -3 1
:3 - integer divide by three -1 -1 0
+2 - add two (call this v) 1 1 3
ʋ - last four links as a dyad, f(v, [F,E])
Ị - insignificant? (abs(x)<=1 ? 1 : 0) 1 1 0
*/ - reduce by exponentiation (i.e. F^E) 2 9 625
, - pair v with that [1,2] [1,9] [3,625]
ị - left (Ị) index into right (that) 1 1 625
*/ - reduce by exponentiation (i.e. F^E) 2 9 625
÷ - divide 1/2 1/9 625/625
P - product 1/18 = 0.05555555555555555




Previous 25 was:



ŒRp`²S⁼ɗƇ⁸+€`Ẏ;Ɗ%³QƊÐLL÷²


Full program brute forcer; possibly longer code than the prime factor route (I might attempt that later).



Try it online!



Starts by creating single moves as coordinates then repeatedly moves from all reached locations accumulating the results, taking modulo n of each coordinate (to restrict to an n by n quadrant) and keeping those which are distinct until a fixed point is reached; then finally divides the count by n^2






share|improve this answer











$endgroup$





















    3












    $begingroup$


    05AB1E, 27 26 25 bytes



    ÓεNØ©<iozë®4%D≠iyÈ®ymz*]P


    Port of @ShieruAsakoto's JavaScript answer, so make sure to upvote him as well!



    Try it online or verify all test cases.



    Explanation:





    Ó                   # Get all prime exponent's of the (implicit) input's prime factorization
    # i.e. 6 → [1,1] (6 → 2**1 * 3**1)
    # i.e. 18 → [1,2] (18 → 2**1 * 3**2)
    # i.e. 20 → [2,0,1] (20 → 2**2 * 3**0 * 5**1)
    # i.e. 25 → [0,0,2] (25 → 2**0 * 3**0 * 5**2)
    ε # Map each value `n` to:
    NØ # Get the prime `p` at the map-index
    # i.e. map-index=0,1,2,3,4,5 → 2,3,5,7,11,13
    © # Store it in the register (without popping)
    <i # If `p` is exactly 2:
    oz # Calculate 1/(2**`n`)
    # i.e. `n`=0,1,2 → 1,0.5,0.25
    ë # Else:
    ®4% # Calculate `p` modulo-4
    # i.e. `p`=3,5,7,11,13 → 3,1,3,3,1
    D # Duplicate the result (the 1 if the following check is falsey)
    ≠i # If `p` modulo-4 is NOT 1 (in which case it is 3):
    yÈ # Check if `n` is even (1 if truthy; 0 if falsey)
    # i.e. `n`=0,1,2,3,4 → 1,0,1,0,1
    ®ymz # Calculate 1/(`p`**`n`)
    # i.e. `p`=3 & `n`=2 → 0.1111111111111111 (1/9)
    # i.e. `p`=7 & `n`=1 → 0.14285714285714285 (1/7)
    * # Multiply both with each other
    # i.e. 1 * 0.1111111111111111 → 0.1111111111111111
    # i.e. 0 * 0.14285714285714285 → 0
    ] # Close both if-statements and the map
    # i.e. [1,1] → [0.5,0.0]
    # i.e. [1,2] → [0.5,0.1111111111111111]
    # i.e. [2,0,1] → [0.25,1.0,1]
    # i.e. [0,0,2] → [1.0,1.0,1]
    P # Take the product of all mapped values
    # i.e. [0.5,0.0] → 0.0
    # i.e. [0.5,0.1111111111111111] → 0.05555555555555555
    # i.e. [0.25,1.0,1] → 0.25
    # i.e. [1.0,1.0,1] → 1.0
    # (and output implicitly as result)





    share|improve this answer











    $endgroup$





















      3












      $begingroup$


      Retina 0.8.2, 126 bytes



      .+
      $*
      +`^(11+)(1)+b
      $1;1$#2$*
      b1(1111)+b
      1
      b(1(11)+);1b
      $1_$1
      .*b(1(11)+)b.*
      0
      _
      ;
      +`G1(?=.*;(1+))|;1+$
      $1
      11+
      1/$.&


      Try it online! Link includes test cases. Uses the prime factorisation. Explanation:



      .+
      $*


      Convert to unary.



      +`^(11+)(1)+b
      $1;1$#2$*


      Get the prime factors.



      b1(1111)+b
      1


      Delete factors of the form 4k+1.



      b(1(11)+);1b
      $1_$1


      Temporarily mark any duplicate pairs of remaining odd factors (i.e. of the form 4k+3).



      .*b(1(11)+)b.*
      0


      If there are any unmatched odd factors remaining then the result is zero.



      _
      ;


      Remove the markings.



      +`G1(?=.*;(1+))|;1+$
      $1


      Multiply any remaining factors back together. (Taken from the Unary arithmetic tutorial.)



      11+
      1/$.&


      Compute the reciprocal as a decimal fraction.






      share|improve this answer









      $endgroup$













        Your Answer





        StackExchange.ifUsing("editor", function () {
        return StackExchange.using("mathjaxEditing", function () {
        StackExchange.MarkdownEditor.creationCallbacks.add(function (editor, postfix) {
        StackExchange.mathjaxEditing.prepareWmdForMathJax(editor, postfix, [["\$", "\$"]]);
        });
        });
        }, "mathjax-editing");

        StackExchange.ifUsing("editor", function () {
        StackExchange.using("externalEditor", function () {
        StackExchange.using("snippets", function () {
        StackExchange.snippets.init();
        });
        });
        }, "code-snippets");

        StackExchange.ready(function() {
        var channelOptions = {
        tags: "".split(" "),
        id: "200"
        };
        initTagRenderer("".split(" "), "".split(" "), channelOptions);

        StackExchange.using("externalEditor", function() {
        // Have to fire editor after snippets, if snippets enabled
        if (StackExchange.settings.snippets.snippetsEnabled) {
        StackExchange.using("snippets", function() {
        createEditor();
        });
        }
        else {
        createEditor();
        }
        });

        function createEditor() {
        StackExchange.prepareEditor({
        heartbeatType: 'answer',
        autoActivateHeartbeat: false,
        convertImagesToLinks: false,
        noModals: true,
        showLowRepImageUploadWarning: true,
        reputationToPostImages: null,
        bindNavPrevention: true,
        postfix: "",
        imageUploader: {
        brandingHtml: "Powered by u003ca class="icon-imgur-white" href="https://imgur.com/"u003eu003c/au003e",
        contentPolicyHtml: "User contributions licensed under u003ca href="https://creativecommons.org/licenses/by-sa/3.0/"u003ecc by-sa 3.0 with attribution requiredu003c/au003e u003ca href="https://stackoverflow.com/legal/content-policy"u003e(content policy)u003c/au003e",
        allowUrls: true
        },
        onDemand: true,
        discardSelector: ".discard-answer"
        ,immediatelyShowMarkdownHelp:true
        });


        }
        });














        draft saved

        draft discarded


















        StackExchange.ready(
        function () {
        StackExchange.openid.initPostLogin('.new-post-login', 'https%3a%2f%2fcodegolf.stackexchange.com%2fquestions%2f179747%2fn-movers-how-much-of-the-infinite-board-can-i-reach%23new-answer', 'question_page');
        }
        );

        Post as a guest















        Required, but never shown

























        6 Answers
        6






        active

        oldest

        votes








        6 Answers
        6






        active

        oldest

        votes









        active

        oldest

        votes






        active

        oldest

        votes









        16












        $begingroup$


        JavaScript (Node.js), 144 138 125 74 73 70 bytes





        f=(x,n=2,c=0)=>x%n?x-!c?f(x,n+1)/(n%4>2?n/=~c&1:n%4)**c:1:f(x/n,n,c+1)


        Try it online!



        -4 byte thanks @Arnauld!



        Original approach, 125 bytes





        a=>(F=(x,n=2)=>n*n>x?[x,0]:x%n?F(x,n+1):[n,...F(x/n,n)])(a).map(y=>r-y?(z*=[,1,.5,p%2?0:1/r][r%4]**p,r=y,p=1):p++,z=r=p=1)&&z


        Try it online!



        Inspired by the video Pi hiding in prime regularities by 3Blue1Brown.



        For each prime factor $p^n$ in the factorization of the number, calculate $f(p^n)$:




        • If $n$ is odd and $pequiv 3text{ (mod 4)}$ - $f(p^n)=0$. Because there is no place to go.

        • If $n$ is even and $pequiv 3text{ (mod 4)}$ - $f(p^n)=frac{1}{p^n}$.

        • If $p=2$ - $f(2^n)=frac{1}{2^n}$.

        • If $pequiv 1text{ (mod 4)}$ - $f(p^n)=1$.


        Multiply all those function values, there we are.



        Update



        Thanks to the effort of contributors from Math.SE, the algorithm is now backed by a proof






        share|improve this answer











        $endgroup$













        • $begingroup$
          Does the video contain a proof? I've been trying to prove this result for a few hours now but I couldn't figure it out.
          $endgroup$
          – infmagic2047
          Feb 11 at 3:38






        • 1




          $begingroup$
          @infmagic2047 Not really, but it gives a method to count the number of points on a $sqrt{n}$ circle. This is useful when coming down to how the n-mover can go.
          $endgroup$
          – Shieru Asakoto
          Feb 11 at 3:39








        • 3




          $begingroup$
          @infmagic2047 I think it's trivial to prove the cases for $q=prod_{pinmathbb{P}land pin{2,3}text{ (mod 4)}}p^{e_p}$ but the cases for the remaining ones are quite hard to prove formally...
          $endgroup$
          – Shieru Asakoto
          Feb 11 at 11:30








        • 1




          $begingroup$
          @infmagic2047 's math.se question about this approach now has an answer with a complete proof.
          $endgroup$
          – Ørjan Johansen
          Feb 13 at 18:09
















        16












        $begingroup$


        JavaScript (Node.js), 144 138 125 74 73 70 bytes





        f=(x,n=2,c=0)=>x%n?x-!c?f(x,n+1)/(n%4>2?n/=~c&1:n%4)**c:1:f(x/n,n,c+1)


        Try it online!



        -4 byte thanks @Arnauld!



        Original approach, 125 bytes





        a=>(F=(x,n=2)=>n*n>x?[x,0]:x%n?F(x,n+1):[n,...F(x/n,n)])(a).map(y=>r-y?(z*=[,1,.5,p%2?0:1/r][r%4]**p,r=y,p=1):p++,z=r=p=1)&&z


        Try it online!



        Inspired by the video Pi hiding in prime regularities by 3Blue1Brown.



        For each prime factor $p^n$ in the factorization of the number, calculate $f(p^n)$:




        • If $n$ is odd and $pequiv 3text{ (mod 4)}$ - $f(p^n)=0$. Because there is no place to go.

        • If $n$ is even and $pequiv 3text{ (mod 4)}$ - $f(p^n)=frac{1}{p^n}$.

        • If $p=2$ - $f(2^n)=frac{1}{2^n}$.

        • If $pequiv 1text{ (mod 4)}$ - $f(p^n)=1$.


        Multiply all those function values, there we are.



        Update



        Thanks to the effort of contributors from Math.SE, the algorithm is now backed by a proof






        share|improve this answer











        $endgroup$













        • $begingroup$
          Does the video contain a proof? I've been trying to prove this result for a few hours now but I couldn't figure it out.
          $endgroup$
          – infmagic2047
          Feb 11 at 3:38






        • 1




          $begingroup$
          @infmagic2047 Not really, but it gives a method to count the number of points on a $sqrt{n}$ circle. This is useful when coming down to how the n-mover can go.
          $endgroup$
          – Shieru Asakoto
          Feb 11 at 3:39








        • 3




          $begingroup$
          @infmagic2047 I think it's trivial to prove the cases for $q=prod_{pinmathbb{P}land pin{2,3}text{ (mod 4)}}p^{e_p}$ but the cases for the remaining ones are quite hard to prove formally...
          $endgroup$
          – Shieru Asakoto
          Feb 11 at 11:30








        • 1




          $begingroup$
          @infmagic2047 's math.se question about this approach now has an answer with a complete proof.
          $endgroup$
          – Ørjan Johansen
          Feb 13 at 18:09














        16












        16








        16





        $begingroup$


        JavaScript (Node.js), 144 138 125 74 73 70 bytes





        f=(x,n=2,c=0)=>x%n?x-!c?f(x,n+1)/(n%4>2?n/=~c&1:n%4)**c:1:f(x/n,n,c+1)


        Try it online!



        -4 byte thanks @Arnauld!



        Original approach, 125 bytes





        a=>(F=(x,n=2)=>n*n>x?[x,0]:x%n?F(x,n+1):[n,...F(x/n,n)])(a).map(y=>r-y?(z*=[,1,.5,p%2?0:1/r][r%4]**p,r=y,p=1):p++,z=r=p=1)&&z


        Try it online!



        Inspired by the video Pi hiding in prime regularities by 3Blue1Brown.



        For each prime factor $p^n$ in the factorization of the number, calculate $f(p^n)$:




        • If $n$ is odd and $pequiv 3text{ (mod 4)}$ - $f(p^n)=0$. Because there is no place to go.

        • If $n$ is even and $pequiv 3text{ (mod 4)}$ - $f(p^n)=frac{1}{p^n}$.

        • If $p=2$ - $f(2^n)=frac{1}{2^n}$.

        • If $pequiv 1text{ (mod 4)}$ - $f(p^n)=1$.


        Multiply all those function values, there we are.



        Update



        Thanks to the effort of contributors from Math.SE, the algorithm is now backed by a proof






        share|improve this answer











        $endgroup$




        JavaScript (Node.js), 144 138 125 74 73 70 bytes





        f=(x,n=2,c=0)=>x%n?x-!c?f(x,n+1)/(n%4>2?n/=~c&1:n%4)**c:1:f(x/n,n,c+1)


        Try it online!



        -4 byte thanks @Arnauld!



        Original approach, 125 bytes





        a=>(F=(x,n=2)=>n*n>x?[x,0]:x%n?F(x,n+1):[n,...F(x/n,n)])(a).map(y=>r-y?(z*=[,1,.5,p%2?0:1/r][r%4]**p,r=y,p=1):p++,z=r=p=1)&&z


        Try it online!



        Inspired by the video Pi hiding in prime regularities by 3Blue1Brown.



        For each prime factor $p^n$ in the factorization of the number, calculate $f(p^n)$:




        • If $n$ is odd and $pequiv 3text{ (mod 4)}$ - $f(p^n)=0$. Because there is no place to go.

        • If $n$ is even and $pequiv 3text{ (mod 4)}$ - $f(p^n)=frac{1}{p^n}$.

        • If $p=2$ - $f(2^n)=frac{1}{2^n}$.

        • If $pequiv 1text{ (mod 4)}$ - $f(p^n)=1$.


        Multiply all those function values, there we are.



        Update



        Thanks to the effort of contributors from Math.SE, the algorithm is now backed by a proof







        share|improve this answer














        share|improve this answer



        share|improve this answer








        edited Feb 14 at 3:21

























        answered Feb 11 at 3:28









        Shieru AsakotoShieru Asakoto

        2,700317




        2,700317












        • $begingroup$
          Does the video contain a proof? I've been trying to prove this result for a few hours now but I couldn't figure it out.
          $endgroup$
          – infmagic2047
          Feb 11 at 3:38






        • 1




          $begingroup$
          @infmagic2047 Not really, but it gives a method to count the number of points on a $sqrt{n}$ circle. This is useful when coming down to how the n-mover can go.
          $endgroup$
          – Shieru Asakoto
          Feb 11 at 3:39








        • 3




          $begingroup$
          @infmagic2047 I think it's trivial to prove the cases for $q=prod_{pinmathbb{P}land pin{2,3}text{ (mod 4)}}p^{e_p}$ but the cases for the remaining ones are quite hard to prove formally...
          $endgroup$
          – Shieru Asakoto
          Feb 11 at 11:30








        • 1




          $begingroup$
          @infmagic2047 's math.se question about this approach now has an answer with a complete proof.
          $endgroup$
          – Ørjan Johansen
          Feb 13 at 18:09


















        • $begingroup$
          Does the video contain a proof? I've been trying to prove this result for a few hours now but I couldn't figure it out.
          $endgroup$
          – infmagic2047
          Feb 11 at 3:38






        • 1




          $begingroup$
          @infmagic2047 Not really, but it gives a method to count the number of points on a $sqrt{n}$ circle. This is useful when coming down to how the n-mover can go.
          $endgroup$
          – Shieru Asakoto
          Feb 11 at 3:39








        • 3




          $begingroup$
          @infmagic2047 I think it's trivial to prove the cases for $q=prod_{pinmathbb{P}land pin{2,3}text{ (mod 4)}}p^{e_p}$ but the cases for the remaining ones are quite hard to prove formally...
          $endgroup$
          – Shieru Asakoto
          Feb 11 at 11:30








        • 1




          $begingroup$
          @infmagic2047 's math.se question about this approach now has an answer with a complete proof.
          $endgroup$
          – Ørjan Johansen
          Feb 13 at 18:09
















        $begingroup$
        Does the video contain a proof? I've been trying to prove this result for a few hours now but I couldn't figure it out.
        $endgroup$
        – infmagic2047
        Feb 11 at 3:38




        $begingroup$
        Does the video contain a proof? I've been trying to prove this result for a few hours now but I couldn't figure it out.
        $endgroup$
        – infmagic2047
        Feb 11 at 3:38




        1




        1




        $begingroup$
        @infmagic2047 Not really, but it gives a method to count the number of points on a $sqrt{n}$ circle. This is useful when coming down to how the n-mover can go.
        $endgroup$
        – Shieru Asakoto
        Feb 11 at 3:39






        $begingroup$
        @infmagic2047 Not really, but it gives a method to count the number of points on a $sqrt{n}$ circle. This is useful when coming down to how the n-mover can go.
        $endgroup$
        – Shieru Asakoto
        Feb 11 at 3:39






        3




        3




        $begingroup$
        @infmagic2047 I think it's trivial to prove the cases for $q=prod_{pinmathbb{P}land pin{2,3}text{ (mod 4)}}p^{e_p}$ but the cases for the remaining ones are quite hard to prove formally...
        $endgroup$
        – Shieru Asakoto
        Feb 11 at 11:30






        $begingroup$
        @infmagic2047 I think it's trivial to prove the cases for $q=prod_{pinmathbb{P}land pin{2,3}text{ (mod 4)}}p^{e_p}$ but the cases for the remaining ones are quite hard to prove formally...
        $endgroup$
        – Shieru Asakoto
        Feb 11 at 11:30






        1




        1




        $begingroup$
        @infmagic2047 's math.se question about this approach now has an answer with a complete proof.
        $endgroup$
        – Ørjan Johansen
        Feb 13 at 18:09




        $begingroup$
        @infmagic2047 's math.se question about this approach now has an answer with a complete proof.
        $endgroup$
        – Ørjan Johansen
        Feb 13 at 18:09











        11












        $begingroup$


        Clean, 189 185 172 171 bytes



        import StdEnv
        $n#r=[~n..n]
        #p=[[x,y]\x<-r,y<-r|x^2+y^2==n]
        =sum[1.0\_<-iter n(q=removeDup[k\[a,b]<-[[0,0]:p],[u,v]<-q,k<-[[a+u,b+v]]|all(e=n>=e&&e>0)k])p]/toReal(n^2)


        Try it online!



        Finds every position reachable in the n-side-length square cornered on the origin in the first quadrant, then divides by n^2 to get the portion of all cells reachable.



        This works because:




        • The entire reachable plane can be considered as overlapping copies of this n-side-length square, each cornered on a reachable point from the origin as if it were the origin.

        • All movements come in groups of four with signs ++ +- -+ --, allowing the overlapping tiling to be extended through the other three quadrants by mirroring and rotation.






        share|improve this answer











        $endgroup$













        • $begingroup$
          My apologies - I was looking at the test cases which go from N=10 to N=13, whereas your test cases include N=11 and N=12 too. You are indeed correct for N=13. +1 from me :)
          $endgroup$
          – trichoplax
          Feb 11 at 0:51






        • 1




          $begingroup$
          @trichoplax I've changed the tests to correspond to the question to avoid the same confusion again
          $endgroup$
          – Οurous
          Feb 11 at 0:53










        • $begingroup$
          I've further tested up to N=145 and all are correct. I couldn't test 146 on TIO due to the 60 second timeout though. I'm expecting some very long run times in answers here...
          $endgroup$
          – trichoplax
          Feb 11 at 1:01






        • 1




          $begingroup$
          Since I took a while to realize this: The reason why the square corners are reachable if there is at least one move (a,b), is the complex equation (a+bi)(a-bi)=a^2+b^2, which in vector form becomes (N,0)=a(a,b)+b(b,-a).
          $endgroup$
          – Ørjan Johansen
          Feb 11 at 17:19
















        11












        $begingroup$


        Clean, 189 185 172 171 bytes



        import StdEnv
        $n#r=[~n..n]
        #p=[[x,y]\x<-r,y<-r|x^2+y^2==n]
        =sum[1.0\_<-iter n(q=removeDup[k\[a,b]<-[[0,0]:p],[u,v]<-q,k<-[[a+u,b+v]]|all(e=n>=e&&e>0)k])p]/toReal(n^2)


        Try it online!



        Finds every position reachable in the n-side-length square cornered on the origin in the first quadrant, then divides by n^2 to get the portion of all cells reachable.



        This works because:




        • The entire reachable plane can be considered as overlapping copies of this n-side-length square, each cornered on a reachable point from the origin as if it were the origin.

        • All movements come in groups of four with signs ++ +- -+ --, allowing the overlapping tiling to be extended through the other three quadrants by mirroring and rotation.






        share|improve this answer











        $endgroup$













        • $begingroup$
          My apologies - I was looking at the test cases which go from N=10 to N=13, whereas your test cases include N=11 and N=12 too. You are indeed correct for N=13. +1 from me :)
          $endgroup$
          – trichoplax
          Feb 11 at 0:51






        • 1




          $begingroup$
          @trichoplax I've changed the tests to correspond to the question to avoid the same confusion again
          $endgroup$
          – Οurous
          Feb 11 at 0:53










        • $begingroup$
          I've further tested up to N=145 and all are correct. I couldn't test 146 on TIO due to the 60 second timeout though. I'm expecting some very long run times in answers here...
          $endgroup$
          – trichoplax
          Feb 11 at 1:01






        • 1




          $begingroup$
          Since I took a while to realize this: The reason why the square corners are reachable if there is at least one move (a,b), is the complex equation (a+bi)(a-bi)=a^2+b^2, which in vector form becomes (N,0)=a(a,b)+b(b,-a).
          $endgroup$
          – Ørjan Johansen
          Feb 11 at 17:19














        11












        11








        11





        $begingroup$


        Clean, 189 185 172 171 bytes



        import StdEnv
        $n#r=[~n..n]
        #p=[[x,y]\x<-r,y<-r|x^2+y^2==n]
        =sum[1.0\_<-iter n(q=removeDup[k\[a,b]<-[[0,0]:p],[u,v]<-q,k<-[[a+u,b+v]]|all(e=n>=e&&e>0)k])p]/toReal(n^2)


        Try it online!



        Finds every position reachable in the n-side-length square cornered on the origin in the first quadrant, then divides by n^2 to get the portion of all cells reachable.



        This works because:




        • The entire reachable plane can be considered as overlapping copies of this n-side-length square, each cornered on a reachable point from the origin as if it were the origin.

        • All movements come in groups of four with signs ++ +- -+ --, allowing the overlapping tiling to be extended through the other three quadrants by mirroring and rotation.






        share|improve this answer











        $endgroup$




        Clean, 189 185 172 171 bytes



        import StdEnv
        $n#r=[~n..n]
        #p=[[x,y]\x<-r,y<-r|x^2+y^2==n]
        =sum[1.0\_<-iter n(q=removeDup[k\[a,b]<-[[0,0]:p],[u,v]<-q,k<-[[a+u,b+v]]|all(e=n>=e&&e>0)k])p]/toReal(n^2)


        Try it online!



        Finds every position reachable in the n-side-length square cornered on the origin in the first quadrant, then divides by n^2 to get the portion of all cells reachable.



        This works because:




        • The entire reachable plane can be considered as overlapping copies of this n-side-length square, each cornered on a reachable point from the origin as if it were the origin.

        • All movements come in groups of four with signs ++ +- -+ --, allowing the overlapping tiling to be extended through the other three quadrants by mirroring and rotation.







        share|improve this answer














        share|improve this answer



        share|improve this answer








        edited Feb 11 at 3:58

























        answered Feb 11 at 0:43









        ΟurousΟurous

        7,05211035




        7,05211035












        • $begingroup$
          My apologies - I was looking at the test cases which go from N=10 to N=13, whereas your test cases include N=11 and N=12 too. You are indeed correct for N=13. +1 from me :)
          $endgroup$
          – trichoplax
          Feb 11 at 0:51






        • 1




          $begingroup$
          @trichoplax I've changed the tests to correspond to the question to avoid the same confusion again
          $endgroup$
          – Οurous
          Feb 11 at 0:53










        • $begingroup$
          I've further tested up to N=145 and all are correct. I couldn't test 146 on TIO due to the 60 second timeout though. I'm expecting some very long run times in answers here...
          $endgroup$
          – trichoplax
          Feb 11 at 1:01






        • 1




          $begingroup$
          Since I took a while to realize this: The reason why the square corners are reachable if there is at least one move (a,b), is the complex equation (a+bi)(a-bi)=a^2+b^2, which in vector form becomes (N,0)=a(a,b)+b(b,-a).
          $endgroup$
          – Ørjan Johansen
          Feb 11 at 17:19


















        • $begingroup$
          My apologies - I was looking at the test cases which go from N=10 to N=13, whereas your test cases include N=11 and N=12 too. You are indeed correct for N=13. +1 from me :)
          $endgroup$
          – trichoplax
          Feb 11 at 0:51






        • 1




          $begingroup$
          @trichoplax I've changed the tests to correspond to the question to avoid the same confusion again
          $endgroup$
          – Οurous
          Feb 11 at 0:53










        • $begingroup$
          I've further tested up to N=145 and all are correct. I couldn't test 146 on TIO due to the 60 second timeout though. I'm expecting some very long run times in answers here...
          $endgroup$
          – trichoplax
          Feb 11 at 1:01






        • 1




          $begingroup$
          Since I took a while to realize this: The reason why the square corners are reachable if there is at least one move (a,b), is the complex equation (a+bi)(a-bi)=a^2+b^2, which in vector form becomes (N,0)=a(a,b)+b(b,-a).
          $endgroup$
          – Ørjan Johansen
          Feb 11 at 17:19
















        $begingroup$
        My apologies - I was looking at the test cases which go from N=10 to N=13, whereas your test cases include N=11 and N=12 too. You are indeed correct for N=13. +1 from me :)
        $endgroup$
        – trichoplax
        Feb 11 at 0:51




        $begingroup$
        My apologies - I was looking at the test cases which go from N=10 to N=13, whereas your test cases include N=11 and N=12 too. You are indeed correct for N=13. +1 from me :)
        $endgroup$
        – trichoplax
        Feb 11 at 0:51




        1




        1




        $begingroup$
        @trichoplax I've changed the tests to correspond to the question to avoid the same confusion again
        $endgroup$
        – Οurous
        Feb 11 at 0:53




        $begingroup$
        @trichoplax I've changed the tests to correspond to the question to avoid the same confusion again
        $endgroup$
        – Οurous
        Feb 11 at 0:53












        $begingroup$
        I've further tested up to N=145 and all are correct. I couldn't test 146 on TIO due to the 60 second timeout though. I'm expecting some very long run times in answers here...
        $endgroup$
        – trichoplax
        Feb 11 at 1:01




        $begingroup$
        I've further tested up to N=145 and all are correct. I couldn't test 146 on TIO due to the 60 second timeout though. I'm expecting some very long run times in answers here...
        $endgroup$
        – trichoplax
        Feb 11 at 1:01




        1




        1




        $begingroup$
        Since I took a while to realize this: The reason why the square corners are reachable if there is at least one move (a,b), is the complex equation (a+bi)(a-bi)=a^2+b^2, which in vector form becomes (N,0)=a(a,b)+b(b,-a).
        $endgroup$
        – Ørjan Johansen
        Feb 11 at 17:19




        $begingroup$
        Since I took a while to realize this: The reason why the square corners are reachable if there is at least one move (a,b), is the complex equation (a+bi)(a-bi)=a^2+b^2, which in vector form becomes (N,0)=a(a,b)+b(b,-a).
        $endgroup$
        – Ørjan Johansen
        Feb 11 at 17:19











        8












        $begingroup$

        Mathematica, 80 bytes



        d[n_]:=If[#=={},0,1/Det@LatticeReduce@#]&@Select[Tuples[Range[-n,n],2],#.#==n&];


        This code is mostly reliant on a mathematical theorem. The basic idea is that the code asks for the density of a lattice given some generating set.



        More precisely, we are given some collection of vectors - namely, those whose length squared is N - and asked to compute the density of the set of possible sums of these vectors, compared to all integer vectors. The math at play is that we can always find two vectors (and their opposite) that "generate" (i.e. whose sums are) the same set as the original collection. LatticeReduce does exactly that.



        If you have just two vectors, you can imagine drawing an identical parallelogram centered at each reachable point, but whose edge lengths are the given vectors, such that the plane is completely tiled by these parallelograms. (Imagine, for instance, a lattice of "diamond" shapes for n=2). The area of each parallelogram is the determinant of the two generating vectors. The desired proportion of the plane is the reciprocal of this area, since each parallelogram has just one reachable point in it.



        The code is a fairly straightforward implementation: Generate the vectors, use LatticeReduce, take the determinant, then take the reciprocal. (It can probably be better golfed, though)






        share|improve this answer











        $endgroup$













        • $begingroup$
          76 bytes: d@n_:=Boole[#!={}]/Det@LatticeReduce@#&@Select[Range[-n,n]~Tuples~2,#.#==n&]
          $endgroup$
          – lastresort
          Feb 11 at 6:00
















        8












        $begingroup$

        Mathematica, 80 bytes



        d[n_]:=If[#=={},0,1/Det@LatticeReduce@#]&@Select[Tuples[Range[-n,n],2],#.#==n&];


        This code is mostly reliant on a mathematical theorem. The basic idea is that the code asks for the density of a lattice given some generating set.



        More precisely, we are given some collection of vectors - namely, those whose length squared is N - and asked to compute the density of the set of possible sums of these vectors, compared to all integer vectors. The math at play is that we can always find two vectors (and their opposite) that "generate" (i.e. whose sums are) the same set as the original collection. LatticeReduce does exactly that.



        If you have just two vectors, you can imagine drawing an identical parallelogram centered at each reachable point, but whose edge lengths are the given vectors, such that the plane is completely tiled by these parallelograms. (Imagine, for instance, a lattice of "diamond" shapes for n=2). The area of each parallelogram is the determinant of the two generating vectors. The desired proportion of the plane is the reciprocal of this area, since each parallelogram has just one reachable point in it.



        The code is a fairly straightforward implementation: Generate the vectors, use LatticeReduce, take the determinant, then take the reciprocal. (It can probably be better golfed, though)






        share|improve this answer











        $endgroup$













        • $begingroup$
          76 bytes: d@n_:=Boole[#!={}]/Det@LatticeReduce@#&@Select[Range[-n,n]~Tuples~2,#.#==n&]
          $endgroup$
          – lastresort
          Feb 11 at 6:00














        8












        8








        8





        $begingroup$

        Mathematica, 80 bytes



        d[n_]:=If[#=={},0,1/Det@LatticeReduce@#]&@Select[Tuples[Range[-n,n],2],#.#==n&];


        This code is mostly reliant on a mathematical theorem. The basic idea is that the code asks for the density of a lattice given some generating set.



        More precisely, we are given some collection of vectors - namely, those whose length squared is N - and asked to compute the density of the set of possible sums of these vectors, compared to all integer vectors. The math at play is that we can always find two vectors (and their opposite) that "generate" (i.e. whose sums are) the same set as the original collection. LatticeReduce does exactly that.



        If you have just two vectors, you can imagine drawing an identical parallelogram centered at each reachable point, but whose edge lengths are the given vectors, such that the plane is completely tiled by these parallelograms. (Imagine, for instance, a lattice of "diamond" shapes for n=2). The area of each parallelogram is the determinant of the two generating vectors. The desired proportion of the plane is the reciprocal of this area, since each parallelogram has just one reachable point in it.



        The code is a fairly straightforward implementation: Generate the vectors, use LatticeReduce, take the determinant, then take the reciprocal. (It can probably be better golfed, though)






        share|improve this answer











        $endgroup$



        Mathematica, 80 bytes



        d[n_]:=If[#=={},0,1/Det@LatticeReduce@#]&@Select[Tuples[Range[-n,n],2],#.#==n&];


        This code is mostly reliant on a mathematical theorem. The basic idea is that the code asks for the density of a lattice given some generating set.



        More precisely, we are given some collection of vectors - namely, those whose length squared is N - and asked to compute the density of the set of possible sums of these vectors, compared to all integer vectors. The math at play is that we can always find two vectors (and their opposite) that "generate" (i.e. whose sums are) the same set as the original collection. LatticeReduce does exactly that.



        If you have just two vectors, you can imagine drawing an identical parallelogram centered at each reachable point, but whose edge lengths are the given vectors, such that the plane is completely tiled by these parallelograms. (Imagine, for instance, a lattice of "diamond" shapes for n=2). The area of each parallelogram is the determinant of the two generating vectors. The desired proportion of the plane is the reciprocal of this area, since each parallelogram has just one reachable point in it.



        The code is a fairly straightforward implementation: Generate the vectors, use LatticeReduce, take the determinant, then take the reciprocal. (It can probably be better golfed, though)







        share|improve this answer














        share|improve this answer



        share|improve this answer








        edited Feb 11 at 3:21

























        answered Feb 11 at 3:09









        Milo BrandtMilo Brandt

        25115




        25115












        • $begingroup$
          76 bytes: d@n_:=Boole[#!={}]/Det@LatticeReduce@#&@Select[Range[-n,n]~Tuples~2,#.#==n&]
          $endgroup$
          – lastresort
          Feb 11 at 6:00


















        • $begingroup$
          76 bytes: d@n_:=Boole[#!={}]/Det@LatticeReduce@#&@Select[Range[-n,n]~Tuples~2,#.#==n&]
          $endgroup$
          – lastresort
          Feb 11 at 6:00
















        $begingroup$
        76 bytes: d@n_:=Boole[#!={}]/Det@LatticeReduce@#&@Select[Range[-n,n]~Tuples~2,#.#==n&]
        $endgroup$
        – lastresort
        Feb 11 at 6:00




        $begingroup$
        76 bytes: d@n_:=Boole[#!={}]/Det@LatticeReduce@#&@Select[Range[-n,n]~Tuples~2,#.#==n&]
        $endgroup$
        – lastresort
        Feb 11 at 6:00











        3












        $begingroup$


        Jelly,  25  24 bytes



        ÆFµ%4,2CḄ:3+2Ịị,*/ʋ÷*/)P


        A monadic link using the prime factor route.



        Try it online!



        How?



        ÆFµ%4,2CḄ:3+2Ịị,*/ʋ÷*/)P - Link: integer, n               e.g. 11250
        ÆF - prime factor, exponent pairs [[2,1], [3,2], [5,4]]
        µ ) - for each pair [F,E]:
        4,2 - literal list [4,2]
        % - modulo (vectorises) [2,1] [3,0] [1,0]
        C - complement (1-x) [-1,0] [-2,1] [0,1]
        Ḅ - from base 2 -2 -3 1
        :3 - integer divide by three -1 -1 0
        +2 - add two (call this v) 1 1 3
        ʋ - last four links as a dyad, f(v, [F,E])
        Ị - insignificant? (abs(x)<=1 ? 1 : 0) 1 1 0
        */ - reduce by exponentiation (i.e. F^E) 2 9 625
        , - pair v with that [1,2] [1,9] [3,625]
        ị - left (Ị) index into right (that) 1 1 625
        */ - reduce by exponentiation (i.e. F^E) 2 9 625
        ÷ - divide 1/2 1/9 625/625
        P - product 1/18 = 0.05555555555555555




        Previous 25 was:



        ŒRp`²S⁼ɗƇ⁸+€`Ẏ;Ɗ%³QƊÐLL÷²


        Full program brute forcer; possibly longer code than the prime factor route (I might attempt that later).



        Try it online!



        Starts by creating single moves as coordinates then repeatedly moves from all reached locations accumulating the results, taking modulo n of each coordinate (to restrict to an n by n quadrant) and keeping those which are distinct until a fixed point is reached; then finally divides the count by n^2






        share|improve this answer











        $endgroup$


















          3












          $begingroup$


          Jelly,  25  24 bytes



          ÆFµ%4,2CḄ:3+2Ịị,*/ʋ÷*/)P


          A monadic link using the prime factor route.



          Try it online!



          How?



          ÆFµ%4,2CḄ:3+2Ịị,*/ʋ÷*/)P - Link: integer, n               e.g. 11250
          ÆF - prime factor, exponent pairs [[2,1], [3,2], [5,4]]
          µ ) - for each pair [F,E]:
          4,2 - literal list [4,2]
          % - modulo (vectorises) [2,1] [3,0] [1,0]
          C - complement (1-x) [-1,0] [-2,1] [0,1]
          Ḅ - from base 2 -2 -3 1
          :3 - integer divide by three -1 -1 0
          +2 - add two (call this v) 1 1 3
          ʋ - last four links as a dyad, f(v, [F,E])
          Ị - insignificant? (abs(x)<=1 ? 1 : 0) 1 1 0
          */ - reduce by exponentiation (i.e. F^E) 2 9 625
          , - pair v with that [1,2] [1,9] [3,625]
          ị - left (Ị) index into right (that) 1 1 625
          */ - reduce by exponentiation (i.e. F^E) 2 9 625
          ÷ - divide 1/2 1/9 625/625
          P - product 1/18 = 0.05555555555555555




          Previous 25 was:



          ŒRp`²S⁼ɗƇ⁸+€`Ẏ;Ɗ%³QƊÐLL÷²


          Full program brute forcer; possibly longer code than the prime factor route (I might attempt that later).



          Try it online!



          Starts by creating single moves as coordinates then repeatedly moves from all reached locations accumulating the results, taking modulo n of each coordinate (to restrict to an n by n quadrant) and keeping those which are distinct until a fixed point is reached; then finally divides the count by n^2






          share|improve this answer











          $endgroup$
















            3












            3








            3





            $begingroup$


            Jelly,  25  24 bytes



            ÆFµ%4,2CḄ:3+2Ịị,*/ʋ÷*/)P


            A monadic link using the prime factor route.



            Try it online!



            How?



            ÆFµ%4,2CḄ:3+2Ịị,*/ʋ÷*/)P - Link: integer, n               e.g. 11250
            ÆF - prime factor, exponent pairs [[2,1], [3,2], [5,4]]
            µ ) - for each pair [F,E]:
            4,2 - literal list [4,2]
            % - modulo (vectorises) [2,1] [3,0] [1,0]
            C - complement (1-x) [-1,0] [-2,1] [0,1]
            Ḅ - from base 2 -2 -3 1
            :3 - integer divide by three -1 -1 0
            +2 - add two (call this v) 1 1 3
            ʋ - last four links as a dyad, f(v, [F,E])
            Ị - insignificant? (abs(x)<=1 ? 1 : 0) 1 1 0
            */ - reduce by exponentiation (i.e. F^E) 2 9 625
            , - pair v with that [1,2] [1,9] [3,625]
            ị - left (Ị) index into right (that) 1 1 625
            */ - reduce by exponentiation (i.e. F^E) 2 9 625
            ÷ - divide 1/2 1/9 625/625
            P - product 1/18 = 0.05555555555555555




            Previous 25 was:



            ŒRp`²S⁼ɗƇ⁸+€`Ẏ;Ɗ%³QƊÐLL÷²


            Full program brute forcer; possibly longer code than the prime factor route (I might attempt that later).



            Try it online!



            Starts by creating single moves as coordinates then repeatedly moves from all reached locations accumulating the results, taking modulo n of each coordinate (to restrict to an n by n quadrant) and keeping those which are distinct until a fixed point is reached; then finally divides the count by n^2






            share|improve this answer











            $endgroup$




            Jelly,  25  24 bytes



            ÆFµ%4,2CḄ:3+2Ịị,*/ʋ÷*/)P


            A monadic link using the prime factor route.



            Try it online!



            How?



            ÆFµ%4,2CḄ:3+2Ịị,*/ʋ÷*/)P - Link: integer, n               e.g. 11250
            ÆF - prime factor, exponent pairs [[2,1], [3,2], [5,4]]
            µ ) - for each pair [F,E]:
            4,2 - literal list [4,2]
            % - modulo (vectorises) [2,1] [3,0] [1,0]
            C - complement (1-x) [-1,0] [-2,1] [0,1]
            Ḅ - from base 2 -2 -3 1
            :3 - integer divide by three -1 -1 0
            +2 - add two (call this v) 1 1 3
            ʋ - last four links as a dyad, f(v, [F,E])
            Ị - insignificant? (abs(x)<=1 ? 1 : 0) 1 1 0
            */ - reduce by exponentiation (i.e. F^E) 2 9 625
            , - pair v with that [1,2] [1,9] [3,625]
            ị - left (Ị) index into right (that) 1 1 625
            */ - reduce by exponentiation (i.e. F^E) 2 9 625
            ÷ - divide 1/2 1/9 625/625
            P - product 1/18 = 0.05555555555555555




            Previous 25 was:



            ŒRp`²S⁼ɗƇ⁸+€`Ẏ;Ɗ%³QƊÐLL÷²


            Full program brute forcer; possibly longer code than the prime factor route (I might attempt that later).



            Try it online!



            Starts by creating single moves as coordinates then repeatedly moves from all reached locations accumulating the results, taking modulo n of each coordinate (to restrict to an n by n quadrant) and keeping those which are distinct until a fixed point is reached; then finally divides the count by n^2







            share|improve this answer














            share|improve this answer



            share|improve this answer








            edited Feb 12 at 2:32

























            answered Feb 11 at 21:48









            Jonathan AllanJonathan Allan

            52.1k535170




            52.1k535170























                3












                $begingroup$


                05AB1E, 27 26 25 bytes



                ÓεNØ©<iozë®4%D≠iyÈ®ymz*]P


                Port of @ShieruAsakoto's JavaScript answer, so make sure to upvote him as well!



                Try it online or verify all test cases.



                Explanation:





                Ó                   # Get all prime exponent's of the (implicit) input's prime factorization
                # i.e. 6 → [1,1] (6 → 2**1 * 3**1)
                # i.e. 18 → [1,2] (18 → 2**1 * 3**2)
                # i.e. 20 → [2,0,1] (20 → 2**2 * 3**0 * 5**1)
                # i.e. 25 → [0,0,2] (25 → 2**0 * 3**0 * 5**2)
                ε # Map each value `n` to:
                NØ # Get the prime `p` at the map-index
                # i.e. map-index=0,1,2,3,4,5 → 2,3,5,7,11,13
                © # Store it in the register (without popping)
                <i # If `p` is exactly 2:
                oz # Calculate 1/(2**`n`)
                # i.e. `n`=0,1,2 → 1,0.5,0.25
                ë # Else:
                ®4% # Calculate `p` modulo-4
                # i.e. `p`=3,5,7,11,13 → 3,1,3,3,1
                D # Duplicate the result (the 1 if the following check is falsey)
                ≠i # If `p` modulo-4 is NOT 1 (in which case it is 3):
                yÈ # Check if `n` is even (1 if truthy; 0 if falsey)
                # i.e. `n`=0,1,2,3,4 → 1,0,1,0,1
                ®ymz # Calculate 1/(`p`**`n`)
                # i.e. `p`=3 & `n`=2 → 0.1111111111111111 (1/9)
                # i.e. `p`=7 & `n`=1 → 0.14285714285714285 (1/7)
                * # Multiply both with each other
                # i.e. 1 * 0.1111111111111111 → 0.1111111111111111
                # i.e. 0 * 0.14285714285714285 → 0
                ] # Close both if-statements and the map
                # i.e. [1,1] → [0.5,0.0]
                # i.e. [1,2] → [0.5,0.1111111111111111]
                # i.e. [2,0,1] → [0.25,1.0,1]
                # i.e. [0,0,2] → [1.0,1.0,1]
                P # Take the product of all mapped values
                # i.e. [0.5,0.0] → 0.0
                # i.e. [0.5,0.1111111111111111] → 0.05555555555555555
                # i.e. [0.25,1.0,1] → 0.25
                # i.e. [1.0,1.0,1] → 1.0
                # (and output implicitly as result)





                share|improve this answer











                $endgroup$


















                  3












                  $begingroup$


                  05AB1E, 27 26 25 bytes



                  ÓεNØ©<iozë®4%D≠iyÈ®ymz*]P


                  Port of @ShieruAsakoto's JavaScript answer, so make sure to upvote him as well!



                  Try it online or verify all test cases.



                  Explanation:





                  Ó                   # Get all prime exponent's of the (implicit) input's prime factorization
                  # i.e. 6 → [1,1] (6 → 2**1 * 3**1)
                  # i.e. 18 → [1,2] (18 → 2**1 * 3**2)
                  # i.e. 20 → [2,0,1] (20 → 2**2 * 3**0 * 5**1)
                  # i.e. 25 → [0,0,2] (25 → 2**0 * 3**0 * 5**2)
                  ε # Map each value `n` to:
                  NØ # Get the prime `p` at the map-index
                  # i.e. map-index=0,1,2,3,4,5 → 2,3,5,7,11,13
                  © # Store it in the register (without popping)
                  <i # If `p` is exactly 2:
                  oz # Calculate 1/(2**`n`)
                  # i.e. `n`=0,1,2 → 1,0.5,0.25
                  ë # Else:
                  ®4% # Calculate `p` modulo-4
                  # i.e. `p`=3,5,7,11,13 → 3,1,3,3,1
                  D # Duplicate the result (the 1 if the following check is falsey)
                  ≠i # If `p` modulo-4 is NOT 1 (in which case it is 3):
                  yÈ # Check if `n` is even (1 if truthy; 0 if falsey)
                  # i.e. `n`=0,1,2,3,4 → 1,0,1,0,1
                  ®ymz # Calculate 1/(`p`**`n`)
                  # i.e. `p`=3 & `n`=2 → 0.1111111111111111 (1/9)
                  # i.e. `p`=7 & `n`=1 → 0.14285714285714285 (1/7)
                  * # Multiply both with each other
                  # i.e. 1 * 0.1111111111111111 → 0.1111111111111111
                  # i.e. 0 * 0.14285714285714285 → 0
                  ] # Close both if-statements and the map
                  # i.e. [1,1] → [0.5,0.0]
                  # i.e. [1,2] → [0.5,0.1111111111111111]
                  # i.e. [2,0,1] → [0.25,1.0,1]
                  # i.e. [0,0,2] → [1.0,1.0,1]
                  P # Take the product of all mapped values
                  # i.e. [0.5,0.0] → 0.0
                  # i.e. [0.5,0.1111111111111111] → 0.05555555555555555
                  # i.e. [0.25,1.0,1] → 0.25
                  # i.e. [1.0,1.0,1] → 1.0
                  # (and output implicitly as result)





                  share|improve this answer











                  $endgroup$
















                    3












                    3








                    3





                    $begingroup$


                    05AB1E, 27 26 25 bytes



                    ÓεNØ©<iozë®4%D≠iyÈ®ymz*]P


                    Port of @ShieruAsakoto's JavaScript answer, so make sure to upvote him as well!



                    Try it online or verify all test cases.



                    Explanation:





                    Ó                   # Get all prime exponent's of the (implicit) input's prime factorization
                    # i.e. 6 → [1,1] (6 → 2**1 * 3**1)
                    # i.e. 18 → [1,2] (18 → 2**1 * 3**2)
                    # i.e. 20 → [2,0,1] (20 → 2**2 * 3**0 * 5**1)
                    # i.e. 25 → [0,0,2] (25 → 2**0 * 3**0 * 5**2)
                    ε # Map each value `n` to:
                    NØ # Get the prime `p` at the map-index
                    # i.e. map-index=0,1,2,3,4,5 → 2,3,5,7,11,13
                    © # Store it in the register (without popping)
                    <i # If `p` is exactly 2:
                    oz # Calculate 1/(2**`n`)
                    # i.e. `n`=0,1,2 → 1,0.5,0.25
                    ë # Else:
                    ®4% # Calculate `p` modulo-4
                    # i.e. `p`=3,5,7,11,13 → 3,1,3,3,1
                    D # Duplicate the result (the 1 if the following check is falsey)
                    ≠i # If `p` modulo-4 is NOT 1 (in which case it is 3):
                    yÈ # Check if `n` is even (1 if truthy; 0 if falsey)
                    # i.e. `n`=0,1,2,3,4 → 1,0,1,0,1
                    ®ymz # Calculate 1/(`p`**`n`)
                    # i.e. `p`=3 & `n`=2 → 0.1111111111111111 (1/9)
                    # i.e. `p`=7 & `n`=1 → 0.14285714285714285 (1/7)
                    * # Multiply both with each other
                    # i.e. 1 * 0.1111111111111111 → 0.1111111111111111
                    # i.e. 0 * 0.14285714285714285 → 0
                    ] # Close both if-statements and the map
                    # i.e. [1,1] → [0.5,0.0]
                    # i.e. [1,2] → [0.5,0.1111111111111111]
                    # i.e. [2,0,1] → [0.25,1.0,1]
                    # i.e. [0,0,2] → [1.0,1.0,1]
                    P # Take the product of all mapped values
                    # i.e. [0.5,0.0] → 0.0
                    # i.e. [0.5,0.1111111111111111] → 0.05555555555555555
                    # i.e. [0.25,1.0,1] → 0.25
                    # i.e. [1.0,1.0,1] → 1.0
                    # (and output implicitly as result)





                    share|improve this answer











                    $endgroup$




                    05AB1E, 27 26 25 bytes



                    ÓεNØ©<iozë®4%D≠iyÈ®ymz*]P


                    Port of @ShieruAsakoto's JavaScript answer, so make sure to upvote him as well!



                    Try it online or verify all test cases.



                    Explanation:





                    Ó                   # Get all prime exponent's of the (implicit) input's prime factorization
                    # i.e. 6 → [1,1] (6 → 2**1 * 3**1)
                    # i.e. 18 → [1,2] (18 → 2**1 * 3**2)
                    # i.e. 20 → [2,0,1] (20 → 2**2 * 3**0 * 5**1)
                    # i.e. 25 → [0,0,2] (25 → 2**0 * 3**0 * 5**2)
                    ε # Map each value `n` to:
                    NØ # Get the prime `p` at the map-index
                    # i.e. map-index=0,1,2,3,4,5 → 2,3,5,7,11,13
                    © # Store it in the register (without popping)
                    <i # If `p` is exactly 2:
                    oz # Calculate 1/(2**`n`)
                    # i.e. `n`=0,1,2 → 1,0.5,0.25
                    ë # Else:
                    ®4% # Calculate `p` modulo-4
                    # i.e. `p`=3,5,7,11,13 → 3,1,3,3,1
                    D # Duplicate the result (the 1 if the following check is falsey)
                    ≠i # If `p` modulo-4 is NOT 1 (in which case it is 3):
                    yÈ # Check if `n` is even (1 if truthy; 0 if falsey)
                    # i.e. `n`=0,1,2,3,4 → 1,0,1,0,1
                    ®ymz # Calculate 1/(`p`**`n`)
                    # i.e. `p`=3 & `n`=2 → 0.1111111111111111 (1/9)
                    # i.e. `p`=7 & `n`=1 → 0.14285714285714285 (1/7)
                    * # Multiply both with each other
                    # i.e. 1 * 0.1111111111111111 → 0.1111111111111111
                    # i.e. 0 * 0.14285714285714285 → 0
                    ] # Close both if-statements and the map
                    # i.e. [1,1] → [0.5,0.0]
                    # i.e. [1,2] → [0.5,0.1111111111111111]
                    # i.e. [2,0,1] → [0.25,1.0,1]
                    # i.e. [0,0,2] → [1.0,1.0,1]
                    P # Take the product of all mapped values
                    # i.e. [0.5,0.0] → 0.0
                    # i.e. [0.5,0.1111111111111111] → 0.05555555555555555
                    # i.e. [0.25,1.0,1] → 0.25
                    # i.e. [1.0,1.0,1] → 1.0
                    # (and output implicitly as result)






                    share|improve this answer














                    share|improve this answer



                    share|improve this answer








                    edited Feb 12 at 8:04

























                    answered Feb 11 at 12:24









                    Kevin CruijssenKevin Cruijssen

                    38.1k557197




                    38.1k557197























                        3












                        $begingroup$


                        Retina 0.8.2, 126 bytes



                        .+
                        $*
                        +`^(11+)(1)+b
                        $1;1$#2$*
                        b1(1111)+b
                        1
                        b(1(11)+);1b
                        $1_$1
                        .*b(1(11)+)b.*
                        0
                        _
                        ;
                        +`G1(?=.*;(1+))|;1+$
                        $1
                        11+
                        1/$.&


                        Try it online! Link includes test cases. Uses the prime factorisation. Explanation:



                        .+
                        $*


                        Convert to unary.



                        +`^(11+)(1)+b
                        $1;1$#2$*


                        Get the prime factors.



                        b1(1111)+b
                        1


                        Delete factors of the form 4k+1.



                        b(1(11)+);1b
                        $1_$1


                        Temporarily mark any duplicate pairs of remaining odd factors (i.e. of the form 4k+3).



                        .*b(1(11)+)b.*
                        0


                        If there are any unmatched odd factors remaining then the result is zero.



                        _
                        ;


                        Remove the markings.



                        +`G1(?=.*;(1+))|;1+$
                        $1


                        Multiply any remaining factors back together. (Taken from the Unary arithmetic tutorial.)



                        11+
                        1/$.&


                        Compute the reciprocal as a decimal fraction.






                        share|improve this answer









                        $endgroup$


















                          3












                          $begingroup$


                          Retina 0.8.2, 126 bytes



                          .+
                          $*
                          +`^(11+)(1)+b
                          $1;1$#2$*
                          b1(1111)+b
                          1
                          b(1(11)+);1b
                          $1_$1
                          .*b(1(11)+)b.*
                          0
                          _
                          ;
                          +`G1(?=.*;(1+))|;1+$
                          $1
                          11+
                          1/$.&


                          Try it online! Link includes test cases. Uses the prime factorisation. Explanation:



                          .+
                          $*


                          Convert to unary.



                          +`^(11+)(1)+b
                          $1;1$#2$*


                          Get the prime factors.



                          b1(1111)+b
                          1


                          Delete factors of the form 4k+1.



                          b(1(11)+);1b
                          $1_$1


                          Temporarily mark any duplicate pairs of remaining odd factors (i.e. of the form 4k+3).



                          .*b(1(11)+)b.*
                          0


                          If there are any unmatched odd factors remaining then the result is zero.



                          _
                          ;


                          Remove the markings.



                          +`G1(?=.*;(1+))|;1+$
                          $1


                          Multiply any remaining factors back together. (Taken from the Unary arithmetic tutorial.)



                          11+
                          1/$.&


                          Compute the reciprocal as a decimal fraction.






                          share|improve this answer









                          $endgroup$
















                            3












                            3








                            3





                            $begingroup$


                            Retina 0.8.2, 126 bytes



                            .+
                            $*
                            +`^(11+)(1)+b
                            $1;1$#2$*
                            b1(1111)+b
                            1
                            b(1(11)+);1b
                            $1_$1
                            .*b(1(11)+)b.*
                            0
                            _
                            ;
                            +`G1(?=.*;(1+))|;1+$
                            $1
                            11+
                            1/$.&


                            Try it online! Link includes test cases. Uses the prime factorisation. Explanation:



                            .+
                            $*


                            Convert to unary.



                            +`^(11+)(1)+b
                            $1;1$#2$*


                            Get the prime factors.



                            b1(1111)+b
                            1


                            Delete factors of the form 4k+1.



                            b(1(11)+);1b
                            $1_$1


                            Temporarily mark any duplicate pairs of remaining odd factors (i.e. of the form 4k+3).



                            .*b(1(11)+)b.*
                            0


                            If there are any unmatched odd factors remaining then the result is zero.



                            _
                            ;


                            Remove the markings.



                            +`G1(?=.*;(1+))|;1+$
                            $1


                            Multiply any remaining factors back together. (Taken from the Unary arithmetic tutorial.)



                            11+
                            1/$.&


                            Compute the reciprocal as a decimal fraction.






                            share|improve this answer









                            $endgroup$




                            Retina 0.8.2, 126 bytes



                            .+
                            $*
                            +`^(11+)(1)+b
                            $1;1$#2$*
                            b1(1111)+b
                            1
                            b(1(11)+);1b
                            $1_$1
                            .*b(1(11)+)b.*
                            0
                            _
                            ;
                            +`G1(?=.*;(1+))|;1+$
                            $1
                            11+
                            1/$.&


                            Try it online! Link includes test cases. Uses the prime factorisation. Explanation:



                            .+
                            $*


                            Convert to unary.



                            +`^(11+)(1)+b
                            $1;1$#2$*


                            Get the prime factors.



                            b1(1111)+b
                            1


                            Delete factors of the form 4k+1.



                            b(1(11)+);1b
                            $1_$1


                            Temporarily mark any duplicate pairs of remaining odd factors (i.e. of the form 4k+3).



                            .*b(1(11)+)b.*
                            0


                            If there are any unmatched odd factors remaining then the result is zero.



                            _
                            ;


                            Remove the markings.



                            +`G1(?=.*;(1+))|;1+$
                            $1


                            Multiply any remaining factors back together. (Taken from the Unary arithmetic tutorial.)



                            11+
                            1/$.&


                            Compute the reciprocal as a decimal fraction.







                            share|improve this answer












                            share|improve this answer



                            share|improve this answer










                            answered Feb 13 at 14:05









                            NeilNeil

                            80.8k744178




                            80.8k744178






























                                draft saved

                                draft discarded




















































                                If this is an answer to a challenge…




                                • …Be sure to follow the challenge specification. However, please refrain from exploiting obvious loopholes. Answers abusing any of the standard loopholes are considered invalid. If you think a specification is unclear or underspecified, comment on the question instead.


                                • …Try to optimize your score. For instance, answers to code-golf challenges should attempt to be as short as possible. You can always include a readable version of the code in addition to the competitive one.
                                  Explanations of your answer make it more interesting to read and are very much encouraged.


                                • …Include a short header which indicates the language(s) of your code and its score, as defined by the challenge.



                                More generally…




                                • …Please make sure to answer the question and provide sufficient detail.


                                • …Avoid asking for help, clarification or responding to other answers (use comments instead).





                                draft saved


                                draft discarded














                                StackExchange.ready(
                                function () {
                                StackExchange.openid.initPostLogin('.new-post-login', 'https%3a%2f%2fcodegolf.stackexchange.com%2fquestions%2f179747%2fn-movers-how-much-of-the-infinite-board-can-i-reach%23new-answer', 'question_page');
                                }
                                );

                                Post as a guest















                                Required, but never shown





















































                                Required, but never shown














                                Required, but never shown












                                Required, but never shown







                                Required, but never shown

































                                Required, but never shown














                                Required, but never shown












                                Required, but never shown







                                Required, but never shown







                                Popular posts from this blog

                                Biblatex bibliography style without URLs when DOI exists (in Overleaf with Zotero bibliography)

                                ComboBox Display Member on multiple fields

                                Is it possible to collect Nectar points via Trainline?